Chuyen de qui nap toan hoc

23 240 0
Chuyen de qui nap toan hoc

Đang tải... (xem toàn văn)

Tài liệu hạn chế xem trước, để xem đầy đủ mời bạn chọn Tải xuống

Thông tin tài liệu

TRƯỜNG ĐẠI HỌC SƯ PHẠM HÀ NỘI KHOA TOÁN TIN ********* Chuyªn ®Ò QUI NẠP TOÁN HỌC Giáo viên hướng dẫn: Đặng Đình Hanh Sinh viên thực hiện: Nguyễn Ngọc Thư Lớp: HK53Toán HÀ NỘI,THÁNG 11-2006 Chuyên đề : Qui nạp toán học NỘI DUNG CHÍNH Phương pháp giải Các dạng toán điển hình Ví dụ minh hoạ Lời giải chi tiết Chú ý Bình luận phân tích Bài tập Chuyên đề : Qui nạp toán học Lời mở đầu Trong khuôn khổ giới hạn chuyên đề nhóm biên soạn xin không đưa khái niệm định nghĩa,mệnh đề, định lí tính chất có SGH phổ thông mà đưa dạng toán kèm theo phương pháp giải , tiếp ví dụ minh họa lời giải chi tiết Kết thúc ví dụ ý cần thiết nhằm tăng chất lượng sư phạm cho chuyên đề Sau dạng toán có đưa loạt tập đề nghị để bạn tham khảo thử sức Khi cần dùng đến kiến thức trình bày lại trước sử dụng giải Mặc dù tham khảo lượng lớn tài liệu với nỗ lực thân trình độ hiểu biết có hạn nên chắn không tránh khỏi thiếu sót mong góp ý thầy giáo Đặng Đình Hanh tập thể lớp K 53H Xin chân thành cảm ơn Một lần nhóm biên soạn xin bày tỏ lòng cảm ơn tới thầy giáo Đặng Đình Hanh cổ vũ, động viên, gợi ý, trình thực chuyên đề này, để chuyên đề sớm hoàn thành Xin chân thành cảm ơn Thầy Chúng xin chân thành cảm ơn bạn Phạm Trà My cung cấp cho nhiều tài liệu hay quý trình thực chuyên đề Thư góp ý bạn xin gửi địa email : tnngocthu@gmail.com Người thực Nguyễn Ngọc Thư Chuyên đề : Qui nạp toán học QUI NẠP Phương pháp qui nạp thực có hiệu lực với lớp toán chứng minh mệnh đề phụ thuộc vào số tự nhiên n ∈ N Phương pháp giải  Để chứng minh mệnh đề Q(n) với n ≥ p , ta thực bước theo thứ tự: Bước : Kiểm tra mệnh đề với n = p Bước : Giả sử mệnh đề với n = k ≥ p , ta phải chứng minh mệnh đề với n = k + Các dạng toán minh hoạ Dạng : Dùng phương pháp qui nạp để chứng minh đẳng thức VD1 : Chứng minh : với số tự nhiên n ≥ ,ta có : an – bn = (a – b)(a n – + a n – 2.b +… +a.b n -2 +b n– ) (1) Ta chứng minh đẳng thức (1) phương pháp qui nạp Giải Khi n=2 VT(1) = a – b , VP(1) = (a –b)(a+ b)= a2 – b2 2 Vậy đẳng thức (1) với n=2 Giả sử (1) với n = k ≥ , tức : a k – b k = (a – b )(a k-1 + a k-2.b + … + a.b k-2 + b k-1 ) Chuyên đề : Qui nạp toán học Ta CM (1) với n=k + , tức : a k+1 – b k+1 = (a-b)(ak + a k-1.b +…+ a.b k-1 + bk) Thật : áp dụng giả thiết qui nạp , ta có : a k+1 - b k+1 = a k+1 – ak.b+ak.b – b k+1 = ak(a-b) + b(ak-bk) = ak(a-b) +b(a-b)(a k-1 + a k-2.b + …+ a.b k-2 +b k-1 ) = (a-b) [ak + b(a k-1 +a k-2 b +…+a.b k-2 +b k-1) ] = (a-b)(ak +a k-1.b +…+a.b k-1 +bk ) Vậy (1) với số tự nhiên n ≥ Bình luận : Trong lời giải ta dùng kĩ thuật thêm bớt số hạng bứơc chứng minh (1) vói n = k+1 ,làm ta sử dụng giả thiết qui nạp toán Đây kĩ thuật hay có hiệu lực mạnh mẽ việc đơn giản hoá lời giải, áp dụng rộng rãi trình giải nhiều dạng toán khác ứng với nhiều chuyên đề khác toán phổ thông Ví dụ sau cho thấy rõ điều (ĐTTS_khối A2002câu  ) Cho phương trình : log 32 x + log 32 x + − 2m − = Giải phương trình (2) m = (2) ( m tham số ) [ Tìm m để phương trình (2) có nghiệm thuộc đoạn 1;3 ] Bình thường không dùng kĩ thuật thêm bớt nhiều học sinh làm sau : Điều kiện x > , Đặt t = log3 x ≥ , pt (2) dạng vô tỉ ,tất nhiên việc giải (2) khó khăn sau hồi lâu cho ta đáp án Tuy nhiên ta thêm đồng Chuyên đề : Qui nạp toán học thời bớt vào vế trái phương trình (2) lại đẳng cấp khác Khi phương trình (2) trở thành : log 32 x + + log 32 x + − 2m − = Điều kiện x > Đặt t = log 32 x + ≥ ta có : t + t − 2m − = (3) Rõ ràng (3) phương trinh bậc biến t, việc giải (3) đơn giản nhanh nhiều so với giải phương trình mà cách đặt mang lại Cũng phải nói thêm có học sinh may mán thấy phương trình có góp mặt thức đặt t thức dẫn tới pt(3) Nhưng may mán ngoại lệ mà số toán mang lại phải kể đến toán Qua phân tích ví dụ ta thấy lợi ích hiệu mà kĩ thuật thêm bớt đem lại cho việc giải toán phổ thông lớn Ta gặp lại kĩ thật lời giải ví dụ (5) sau số ví dụ khác có mặt chuyên đề Xin mời bạn theo dõi VD2: CMR : Mọi số tự nhiên n ≥ , ta có : 12 + 2 + + + ( n − 1) + n = n( n + 1)( 2n + 1) (2) Giải Khi n = VT(2) = VP(2) nên (2) Giả sử (2) với n = k ≥ , tức : 12 + 22 + 32 + + ( k − 1) + k2 = k(k + 1)(2k + 1) Ta phải chứng minh (2) với n = k +1 , tức : 12 + 2 + + + [ ( k + 1) − 1] + ( k + 1) = Thật : (k + 1)(k + 2)(k + 3) 12 +22+32+…+(k-1)2 + k2 +(k+1)2 Chuyên đề : Qui nạp toán học 2 = 1 + + + + ( k − 1) + k  + ( k + 1) = 2 k (k + 1)(2k + 1) + (k+1)2  2k2 + 7k + 6 = ( k + 1)     = (k + 1)(k + 2)(2k + 3) • Vậy (1) với số tự nhiên n ≥ Chú ý : lời giải đặc biệt kĩ nhóm số hạng tinh tế để thành lập xuất giả thiết qui nạp bước n = k+1 dẫn đến giải toán VD3 Tìm số hạng tổng quát dãy số sau : u1 = 3, u n +1 = 2u n , ( n ≥ 1) Giải Ta có : u1 = = 3.20 u2 = 2.u1 = 2.3 = = 3.21 u3 = 2.u2 = 2.6 = 12 = 3.22 un = 3.2n −1 n −1 Ta chứng minh un = 3.2 (3) qui nạp ( dogt ) Khi n = ta có u1 = → (3) k −1 Giả sử (3) với n = k, ( k ≥ 1) tức : uk = 3.2 k Ta phải chứng minh (3) với n = k+1 , tức : uk +1 = 3.2 k −1 k Thật : uk +1 = 2.uk = 3.2.2 = 3.2 Vậy (3) với n = k+1 nên vơi n ≥ • Chuyên đề : Qui nạp toán học Chú ý : Sau ví dụ ba ta rút phương pháp giải chung cho dạng toán tìm số hạng tổng quát dãy số gồm hai bước :  Bước : Tìm vài số hạng đầu dãy  Bước : Dự đoán số hạng tổng quát, chứng minh qui nạp y= VD4: Tính đạo hàm cấp n hàm số sau : 1+ x Giải y, = − Ta có : (1 + x) y ,, = , − y ,, , = (1 + x) , (1 + x) ,…, y (n ) (n ) Bây ta tìm y quy nạp sau : Giả sử y(k) = y ( k +1) Ta có : yn = Vậy • ( − 1) k k! (1 + x ) k +1 =y (k ) ,  (−1)(k + 1)(1 + x) = [( − 1) k!]. (1 + x) k ( k +1)  k  (−1) k +1 (k + 1)! = (1 + x) ( k +1) +1  (−1).n! (1 + x) n +1 Chú ý : Phương pháp giải chung cho dạng toán phân làm hai bước sau :  Bước : Tính đạo hàm cấp , hai,ba,…,cho tới dự đoán đạo hàm cấp n  Bước 2: Chứng minh đạo hàm cấp n qui nạp toán học VD5 : (Đề thi học kì 1, Đại số tuyến tính - lớp K53GH_2003) CMR : Nếu số phức z thỏa mãn : 1 + z = cos α ⇒ n + z n = cos α z z Giải (5) Chuyên đề : Qui nạp toán học Với n=1, VT (5) = +z z , VP(5)= cos α theo giả thiết (5) Giả sử (5) với n=k , tức : + z k = cos kα k z Ta phải chứng minh (5) với n=k+1, tức : Thật : z k +1 z k +1 + z k +1 = cos ( k + 1) α      + z k +1 =  k + z k ÷ + z ÷−  z k −1 + k −1 ÷ z  z  z   = cos kα cos α − cos( k − 1).α [ cos(k − 1)α + cos(k + 1)α ] − cos ( k − 1) α =2cos(k+1) α Vậy (5) với n = k +1,nên (5) với ∀n ≥ = • Chú ý : không bình luận thêm lời giải Thật bất ngờ lại đề thi học kì cấp độ đại học Điều chứng tỏ qui nạp vấn đề nguội lạnh kì thi.Do việc nắm vững phương pháp giải điều thật cần thiết với người học làm toán Bình luận chung cho dạng : Qua năm ví dụ ta thấy toán chứng minh đẳng thức cách dùng phương pháp qui nạp toán học khó khăn phức tạp phần cuối bước , tức chứng minh đẳng thức với n=k+1.Khi từ đẳng thức cần chứng minh ứng với n=k+1,ta biến đổi khéo léo,(dùng kĩ thuật thêm bớt ,hoặc tách số hạng… ), để sử dụng giả thiết đẳng thức với n=k,tiếp tục thực tính toán số bước ta có Đpcm Cần nhấm mạnh với dạng toán ta thường biến đổi theo đưòng ! Tuy nhiên cách biến đổi nhất,ta biến đổi trực tiếp từ giả thiết đẳng thức với n = k (giả thiết qui nạp toán) , để suy đẳng thức với n = k+1 Để minh hoạ cho cách làm ta xét ví dụ sau : n 2n + + + + n = − CMR n thuộc N* ta có : (BL) 3 4.3 n Giải Chuyên đề : Qui nạp toán học Với n = , (BL) : = − 12 Giả sử (BL) với n = k, tức : k 2k + + + + k = − (BL.1) 3 4.3k Ta phải chứng minh (BL) với n = k+1, tức : k ( k + 1) 2( k + 1) + + + + k + k +1 = − 3 3 4.3 k +1 (BL.2) Thật : Cộng vào hai vế (BL.1) lượng : k +1 , ta (BL.2) 3k +1 Vậy (BL) với n = k+1, nên với n thuộc N* Kĩ thuật biến đổi lần thể ví dụ (8) dạng hai qui nạp toán học Xin mời bạn theo dõi Bài tập đề nghị Bài 1: CMR : Mọi n ∈ N * , ta có : 1+3+5+…+(2n-1) = n2 Bài : CMR: ∀n ∈ N * , ta có : + + + + n = Bài : CMR : Mọi n ∈ N * ,ta có : 13 + + + n = Bài : CMR : n ( n + 1) n ( n + 1) Mọi a >0, a ≠ 1, x1 , x2 , , xn > ,ta có hệ thức sau: log a ( x1 x2 xn ) = log ax1 + log ax2 + + log axn CMR: Mọi số tự nhiên n ≥ 1, với cặp số (a,b),ta có công thức sau đây, gọi công thức khai triển nhị thức niutơn (a+b) n =C0n a n +C1n a n-1b1+C 2n a n-2b 2+ +C kn a n-k b k + +C nn b n  n ( n + 1)  Bài 6: CMR : s n = 13 + 23 + 33 + + n3 =     Bài 5: Bài 7: Bài 8: CMR: Với số tự nhiên n ≥ 1,ta có đăng thức : n(n + 1) + + + + n = CMR : Mọi n thuộc N ta có : ( − 10    ) 1 − 1 −  1 −   25   ( 2n − )  + 2n =  − 2n  Chuyên đề : Qui nạp toán học Bài 9: Tính đạo hàm cấp n hàm số sau : a) y = ln(1 + x ) c) y = sin ax s n = y= b) ( a = const ) x ( 1− x) d) y = sin x 1 + + + 1.2 2.3 n( n + 1) Bài 10: Tìm tổng số Bài 11: Tìm số hạng tổng quát dãy số sau : a) u1 = 3, un +1 = + un b) u1 = a, un +1 = a + b.un • Các tập đề nghị đưa lựa chọn cẩn thận, kĩ lưỡng, phần có tính chất định hướng phân loại theo loại toán chữa dạng Dạng 2: Dùng phương pháp qui nạp để chứng minh bất đẳng thức VD1: Chứng minh bất đẳng thức Bec-nu-li(Bernoulli) Nếu h >0 , với số tự nhiên n ≥ (1 + h) n > + nh (1) , Giải Nếu n =2, ta có : (1+h) = 1+2h+h > 1+2h (vì h2 > 0) Vậy (1) Giả sử (1) đến n = k , tức :( 1+h)k > 1+kh (2) Ta phải chứng minh (1) đến n =k+1 ,tức : (1+h)k+1 > 1+(k+1)h Thật : (1+h)k+1 =(1+h)(1+h)k ( 2) ≥ (1+h)(1+kh) =1+h+kh+kh2 = 1+h(1+k)+kh2 > 1+h(1+k).(vì kh2 >0) Vậy (1) với số tự nhiên n ≥ • Chú ý : Phép chứng minh giả thiết h không phụ thuộc n Trong trường hợp h phụ thuộc n , người ta chứng minh bất đẳng thức bec_nu_li (dùng công thức nhị thức niutơn ) VD2 : (ĐỀ 101 câu 4a_BĐTTS) Chứng minh x >0 với số tự nhiên n ta có : x2 xn e > 1+ x + + + 2! n! x (2) 11 Chuyên đề : Qui nạp toán học Giải Xét hàm số f  x2 xn  ( x ) = e − 1 + x + + + ÷ n 2! n!   x f ( x) > phải chứng minh : ∀x > 0, n ∈ N : Thật , ta có : ∀n, n Ta (2.1) f ( 0) = n Xét f ( x) = e − ( 1+ x) Ta có f ( x) = e x , x − > 0, ( ∀x > ) , f ( x) ⇒ f ( x ) > f ( 0) tăng với x >0 1 Vậy công thức (2.1) với n=1 Giả sử bất đẳng thức với n=k Ta có: f ( x) = e ∀x > 0, k x  x2 xk  − 1 + x + + + ÷ > (2.2) 2! k!   Ta phải chứng minh : ∀x > 0, f  x2 xk xk+1  x x = e − + x + + + + >0 ( )  ÷ ÷ k+1 2! k ! k + ! ( )   k  2x k.xk−1 ( k + 1) x  x x = e − + + + + : f k+1 ( )  ÷ 2! k! ( k + 1) ! ÷  , Thật , ta có  , x k −1 xk  f k +1 ( x ) = e x − 1 + x + + k − + k ! ÷ = f k ( x )  Theo (2.2) có ∀x > ⇒ f ( x) > ⇒ f k , k +1  ( x ) > ⇒ f k +1 ( x ) tăng với f ( x ) > f ( 0) = k +1 k +1 Vậy bất đẳng thức với n=k+1 nên với số tự nhiên n • Chú ý : Nhìn vào bđt (2) ta thấy hai vế hàm số biến x Nếu ta chuyển toàn vế phải bđt (2) sang vế trái đặt : ∀x > 0, ∀n ∈ N , f ( x ) toán trở thành Cmr n f ( x ) > Khi dùng qui nạp để xử lí toán kết hợp với ứng dụng n đạo hàm tính đơn điệu hàm số vô hợp lí.Rõ ràng điểm mẫu chốt,bước đột phá đưa đến hướng giải đẹp cho toán thao tác chuyển vế 12 Chuyên đề : Qui nạp toán học VD3 (ĐỀ131CÂU4a_BĐTTS) : Cho hàm số f xác định với x thoả mãn điều kiện : f(x+y) ≥ f(x).f(y) với x,y (3) CMR : Với số thực x số tự nhiên n ta có :   x f ( x) ≥ f  n  2 2n    (3.1) Giải x Trong BĐT f(x+y) ≥ f(x).f(y) thay x y , ta được: x f + 2 x x   x  x  )  ≥ f   f   ⇒ f ( x ) ≥  f ( 2  2 2    x f ( x) ≥  f  n  2 Vậy bất đẳng thức    2n Giả sử bất đẳng thức với n =k , với n=1 ( k ≥ 1)   x f ( x) ≥  f  k  2 Ta có      x  f ( x ) ≥  f  k +1     Ta chứng minh bất đẳng thức với n = k+1, tức : Thật ta có :  x   x x    x  f = f + ÷ ÷≥  f  ÷  2k   2k + 2k +    2k +   k 2k  2   x    x   ⇒f   ≥   f    ÷ k +1 ÷   2k             x  ⇒f  ÷   2k   2k   x  ≥f  ÷   2k +   2k +   x  f ( x ) ≥  f  k +1     Do tính chất bắc cầu ta có : k +1 Bất đẳng thức với n = k+1 nên với số tự nhiên n 13 2k k +1 • Chuyên đề : Qui nạp toán học Chú ý : ta gặp dạng toán chứng minh BĐT (a) BĐT (b) xảy Nói cách khác BĐT (a) xảy có BĐT (b) Hướng giải giành cho dạng xuất phát từ BĐT (b) để chứng minh BĐT (a) Đúng Thực chất toán VD3 chứng minh (3.1) với số thực x,mọi số tự nhiên n hàm f thỏa mãn (3).Do dùng qui nạp để chứng minh (3.1) tiến hành điều kiện buộc hàm f sử dụng tính chất bắc cầu VD4 : ∀n ∈ N * ,ta có : sin nα ≤ n sin α (4) Với n =1 , VT (4) = sin1.α = sin α = VP (4) nên (4) Giả sử (4) với n = k ( k ≥ 1) , với n = k+1,tức : Thật vậy, ta có tức : sin kα ≤ k sin α Ta phải chứng minh (4) sin( k + 1)α ≤ ( k + 1) sin α sin ( k + 1) α = sin kα cos α + cos kα sin α ≤ sin kα cos α + cos kα sin α ≤ sin kα + sin α ≤ k sin α + sin α ≤ ( k + 1) sin α Vậy (4) với n = k+1 , nên (4) với số nguyên dương n • Chú ý : Ta thấy (4) có chứa hàm lượng giác nên việc chứng minh (4) qui nạp thực tính chất hàm lượng giác ,cụ thể ta dùng công cộng hàm sin , tính chất hàm sin , cos, nhận giá trị đoạn [ − 1,1] bất đẳng thức a + b ≤ a + b VD5 : Chứng minh dãy số sau giảm bị chặn u1 = 2, un +1 = un + với ( ∀n ≥ 1) Giải Chứng minh dãy số giảm Ta dùng qui nạp Ta phải chứng minh : ( un +1 < un , ∀n ∈ N * Khi n = u2 = ) (5) u1 + + = = < = u1 (5) 2 Giả sử (5) với n = k , ( k ≥ 1) , tức : 14 uk +1 < uk (5.1) Chuyên đề : Qui nạp toán học Ta phải chứng minh : uk + < uk +1 Ta có : uk + = uk +1 + ( 5.1) uk + < = uk +1 * Vậy (5) với n = k+1 nên với n thuộc N Chứng minh dãy cho bị chặn Ta dùng qui nạp để chứng minh : ( ) u > 1, ∀n ∈ N * (6) n Khi n=1 , u1 = > nên (6) Giả sử (6) với n = k , ( k ≥ 1) nghĩa uk > (6.1) Ta phải chứng minh : Ta có : • u k +1 = uk +1 > uk + 1 + > =1 2 Vậy uk +1 > Dãy số cho bị chặn Chú ý : Khi gặp dạng toán chứng minh dãy số đơn điệu bị chặn ta thực sau :  bước : Dùng qui nạp để chứng minh dãy số đơn điệu  bước : Dự đoán số M trường hợp dãy bị chặn M Số m trường hợp ngược lại.Sau dùng qui nạp để chứng minh dãy bị chặn M bị chặn m trường hợp ngược lại VD 6: n  1 Chứng minh : 1 +  < n, ∀n ∈ N , n > (6)  n Giải 64  1 Khi n =3 bđt (6) trở thành 1 +  = < (đúng) 27  3 k Giả sử bđt (6) với n =k nghĩa :  1 1 +  < k  k    < k +1 Ta chứng minh bđt (6) với n = k + , tức : 1 +  k + 1   1 +  Ta có :  k +  k +1 k    ( 2)   = 1 +  1 +  < 1 +  k +1  k + 1  k    ( gtqn )  k 1 +  1 +  k  k <  Vậy bđt(6) với n= k+1 nên với n 15 1  = k +1 k • Chuyên đề : Qui nạp toán học Chú ý : lời giải ta dùng phương pháp làm trội đánh giá bđt bước n =k +1,tại vị trí dấu bđt (2).Có thể nói phương pháp chủ công, mang tính đặc thù chứng minh bđt Học sinh cần nắm vững làm tốt phương pháp hiệu mà mang lại, lưu ý không nên đánh giá bđt lỏng , chặt Sau ví dụ minh hoạ giành cho phương pháp đánh giá làm trội VD 7: Cho x1,x2,…,xn số dương Chứng minh : x n −1 xn x1 x2 x3 + + + + + ≥ 2, n ≥ x + x n x3 + x1 x + x x n + x n − x1 + x n −1 (7) Giải Với n = , bđt có dạng : x3 x + x3 x + x x1 x2 x4 + + + ≥2⇔ + ≥2 x + x x3 + x1 x + x x1 + x3 x + x x1 + x3 Giả sử bđt(7) với n = k Tức : x k −1 xk x1 x2 + + + + ≥ 2, ( k ≥ 4) (7.2) x + x k x3 + x1 x k + x k − x1 + x k −1 Ta chứng minh bđt(7) với n = k+1 Do vai trò bình đẳng giữ xi ( i = 1,2,…,k+1), nên không giảm tính tổng quát toán ta giả sử xk+1 = min{ x1,x2,…,xn } , tức : xk + > 0, xk + ≤ xk , xk + ≤ x1 Do ta có : s k +1 = xk x xk x1 x2 x1 x2 + + + + k +1 > + + + x + x k +1 x3 + x1 x k +1 + x k −1 x1 + x k x + x k x3 + x1 x1 + x k −1 (7.1) Do: xk xk x x1 x1 ≥ ; ≥ ; k +1 > (7.3) x + x k +1 x + x k x k +1 + x k −1 x1 + x k −1 x1 + x k Từ (7.1),(7.2),(7.3) suy s k +1 > Vậy bđt với n = k + nên với n Đó Đpcm • Chú ý : Thí dụ cho thấy rõ nét sức mạnh phương pháp đánh giá làm trội chứng minh bđt Bước ngoặt đưa đến hướng giải cho lời giải toán thao tác đánh giá , ước lượng , giá trị xk+1 = minxi,{ i= 1,2,…n} bước n = k+1 VD : Chứng minh : ∀n ≥ , ta có 16 1.3.5 ( 2n − 1) < 2.4.6.2n 2k + (8) Chuyên đề : Qui nạp toán học Khi n = , (1) trở thành : 1 < Giả sử (8) vớii n = k ,nghĩa : 1.3.5 ( 2k − 1) < 2.4.6 2n Ta phải chứng minh (8) với n = k+1, tức : Thật , ta có : ( 2k + 1) < 2k + 2k + 2k + (8.2) 2k + (8.1) 1.3.5 ( 2k − 1)( 2k + 1) < 2.4.6 2k ( 2k + ) 2k + ( bđt (8.2) sau bình phương hai vế , quy đồng , chuyển vế ta thu bđt tương đương : > ,đúng ) Lấy (8.1) nhân (8.2) vế theo vế ta\có : 1.3.5 ( 2k − 1)( 2k + ) < 2.4.6 2k ( 2k + ) 2k + 2k + 2k + = 2k + Theo nguyên lí qui nạp ta kết luận (8) ∀n ≥  Chú ý : ví dụ (8) minh chứng lại lần cho kĩ thuật sử dụng trực tiếp giả thiết qui nạp toán (giả thiết bđt với n =k ) để thực biến đổi suy bđt với n = k+1 lời giải ví dụ (8) ví dụ (BL) dạng cho thấy biến đổi từ đt, bđt ứng với n = k+1, để dùng giả thiết qui nạp dẫn đến kết thúc toán mà ta biến đổi trực tiếp từ giả thiết qui nạp toán Đối với toán qui nạp để linh hoạt trình giải ta nên nhớ hai cách làm Vẫn nói thêm hai vd(8) vd(BL) hoàn toàn giải cách biến đổi từ đt, bđt ứng với n = k +1 ♦ Bình luận chung cho dạng : Qua tám ví dụ ta thấy toán dạng hai phong phú , đa dạng nhiều so với dạng , độ khó tăng lên Do việc nắm vững cách giải chưa đủ để giải toán.Rõ ràng mẫu chốt toán kĩ thuật biến đổi bất đẳng thức ứng với n=k+1 để sử dụng giả thiết bất đẳng thức với n = k,hoặc biến đổi trực tiếp từ bất đẳng thức với n= k (đây gọi giả thiết qui nạp) để suy bất đẳng thức với n = k+1 Khi việc đến điều phải chứng minh khó khăn Bài tập đề nghị Bài Cho < α < π ( n − 1) Chứng minh : tgnα > ntgα 17 Chuyên đề : Qui nạp toán học a + a + + a < Bài Chứng minh : với a >0 Bài Chứng minh : n n +1 + 4a + > ( n + 1) , ( n ≥ 3) n Bài Chứng minh với số tự nhiên n ta có : a )1 + + + + n ≤ n b)1 + c)1 + Bài + + + n n +1 n ≤ n −1 1 n + + + > −1 Chứng minh bất đẳng thức : (1 + )(1 + 2 22 )(1 + ) × × (1 + ) < 13 23 2n n +1 Bài Chứng minh với số nguyên dương n , ta có : 2n − 1 a) < 46 2n 2n + 2n b)( 2n!< ( n!) ) Bài Chứng minh : Bài Chứng minh dãy số u n xác định : n + + + n < , ∀n ∈ N 3 u1 = 2,u n +1 = + u n , ∀n ∈ N * tăng bị chặn Bài CMR n n +1 = ( n + 1) n Bài 10 1 Chứng minh số tự nhiên n khác ta có : ≤   < n Bài 11 n n với số tự nhiên n lớn ta có :   > n! >   2 3 với n ≥3 n n CMR Bài 12 CMR : Bài 13 ( 2n )! ≥ 4n , n ∈ N * ( n!) n + Cho n số dương nghiệm điều kiện a1 a a n = CMR : a1 + a + + a n ≥ n(*) Dấu ‘’=’’xảy ? 18 n Chuyên đề : Qui nạp toán học Bài 14 Chứng minh số tự nhiên n >1, ta có : ( n + 1) cos Bài 15 Cho n số tự nhiên ( )( π π − n cos > n +1 n π < ( n + 1) α < ) CMR : − cos n α + cos n α < tgnα tgα Bài 16 • ∀n ∈ N , n > 1.CMR : 1 13 + + > n +1 n + 2n 14 Qua hai dạng đầu qui nạp toán học ta có cảm giác mức độ hay va khó toán tăng dần.Do đặc thù ,hai dạng học tương đối sâu phổ thông.Dạng ba toán ,cũng dạng cuối trình bày chuyên đề học sơ qua bậc phổ thông học cao năm thứ hai trường Đhsp Cũng lí mà dạng ba đưa vào sau Xin mời bạn chuyển sang dạng ba qui nạp toán học • Dạng : Dùng qui nạp toán học để chứng minh biểu thức dạng Un chia hết cho số tự nhiên * VD1: Chứng minh ∀n ∈ N , a n = n + 3n + 5n chia hết cho (1) Giải Với n = ta có : a1 = 13 + 3.12 + 5.1 = 3 Giả sử (1) với n = k , ( k ≥ 1) , tức : a k = k + 3k + 5k 3 Ta phải chứng minh (1) với n = k+1, nghĩa : a k +1 = ( k + 1) + 3( k + 1) + 5( k + 1) 3 3 2 Thật : a k +1 = k + 3k + 3k + + 3k + 6k + + 5k + = k + 3k 2 +5k + 3k 2 + 9k + 3 3 3 Vậy (1) với n = k+1, nên với n ∈ N * • Chú ý : Ta biết tổng chia hết cho số số hạng tổng chia hết cho số Nhận thấy a k +1 tổng đa thức k , Vậy để chứng minh a k+1 chia hết cho ta phải thác triển a k+1, sau tiến hành thực xếp lại số hạng , kết hợp với giả thiết qui nạp , viết lại ak+1 dạng tổng số hạng chia hết cho VD2:Chứng minh ∀n ≥ , ta có : an = Giải 19 ( n + 1)( n + 2) ( n + n ) 2 n (2) Khi n = , ta có : a2 = ( + 1)( + ) 2 Chuyên đề : Qui nạp toán học Giả sử (2) với n =k , ( k ≥ ) , tức : ak = ( k + 1)( k + ) ( k + k ) 2 k Ta phải chứng minh (2) với n = k+1, nghĩa : ( k + + 1)( k + + 2) ( k + k + + 1) 2 k +1 = ( k + 2)( k + 3) ( k + k + ) = ( k + 2)( k + 3) ( k + k )( k + k + 1)( k + k + ) ak+1 = = ( k + 1) ( k + ) ( k + 3) ( k + k )  ( k + k + 1) M2k + 1 4 4 42 4 4 43 42 43 M2 M2k Vậy (2) với n = k+1 ,nên (2) với ∀n ≥ • Chú ý : Lời giải ví dụ hai lạ , ta thực kĩ thuật viết lại a k+1 , thành lập xuất giả thiết qui nạp , dễ dàng suy đpcm VD3: Chứng minh : an = 33n +3 − 26n − 27 676, ∀n ≥ (3) Giải Với n = , ta có :a1 = 33.1+3 − 26.1 − 27 = 676 676 nên (3) Giả sử (3) với n = k , k ≥ tức : ak = 33k +3 − 26k − 27 676 (3.1) Ta phải chứng minh (3) với n = k+1, tức : ak+1= 33( k +1) +3 − 26.( k + 1) − 27 676 Thật :ak+1 = ( ) 33( k +1) +3 − 26(k + 1) − 27 = 27 33k + − 26k − 27 + 676   k +676  Vậy (3)        676 676 ( 3.1) với n = k+1 , nên (3) với n ≥ • Chú ý : ví dụ việc viết lại a k+1 không đơn thác triển xếp lại số hạng , rõ ràng kĩ thuật thêm bớt lại phát huy tác dụng, việc đưa 27 làm thừa số chung, với mục đích thành lập gtqn làm dư lượng so với lượng ban đầu , để cân toán ta thêm vào lượng 676k+676 Làm ta dùng gtqn tiến đến kết thúc lời giải Ví dụ bốn minh học cho lời giải loại tập VD4: Chứng minh rằng: ∀n ≥ : n −1.2 n +1 + n +1.2 n −1 38 (4) Với n = , ta có : 2.1−1.21+1 + 31+1.2 2.1−1 = 5.4 + 9.2 = 3838 nên (4) Giả sử (4) với n = k, ( k ≥ 1) tức : k −1.2 k +1 + k +1.2 k −1 38 Ta phải chứng minh (4) với n = k+1, tức : 20 (4.1) Chuyên đề : Qui nạp toán học k + −1 k +1+1 +3 k +1+1 2 k + −1 38 Thật : k + 2−1.2 k +1+1 + k +1+1.2 k + 2−1 = 25.5 k −1.2.2 k +1 + 3.3 k +1.4.2 k −1 = 50.5 k −1.2 k +1 + 12.3 k +1 2 k −1 ( ) k −1 = 50 k −1.2 k +1 + k +1.2 k −1 − 38.3 k +1.2  38          38 38 ( 4.1) Vậy (4) với n = k+1 , nên với ∀n ∈ N * VD5: Chứng minh : ∀n ≥ , ta có : 3n − 14n + 21n − 10n 24 (5) Giải Với n = ta có : − 14 + 21 − 10 = 24 , nên (5) Giả sử (5) với n = k, k ≥ , nghĩa : 3k − 14k + 21k − 10k 24 (5.1) Ta phải chứng minh (5) vói n = k+1 , nghĩa : 3( k + 1) − 14( k + 1) + 21( k + 1) − 10( k + 1) 24 Thật : ( ) ( ) 3( k + 1) − 14( k + 1) + 21( k + 1) − 10( k + 1) = k + 4k + 6k + 4k + − 14 k + 3k + 3k + ( ) + 21 k + 2k + − 10( k + 1) = 3k − 14k + 21k _ 10k + 12.k ( k − 1)( k − 1) 24                24 ( 5.1) 24 Vậy (5) với n = k+1 , nên với ∀n ≥ • Chú ý : Ví dụ ví dụ thuộc dạng Do cách giải giành cho ví dụ xem ý ví dụ ♦ Bình luận chung cho dạng 3: Qua năm ví dụ giành cho dạng ba ta thấy mẫu chốt để giải tốt tập dạng ba kĩ viết lại a n ứng với n = k+1,thành tổng số hạng tích thừa số chia hết cho số tự nhiên cần chứng minh Tất nhiên trình viết lại vậy, ta lưu ý tới việc sử dụng giả thiết qui nạp toán.Có thể nói kĩ thuật viết lại đề toán nói riêng viết lại biểu thức toán học nói chung , để dùng giả thiết toán , đặc biệt có hiệu quả, giải toán phổ thông Xin đưa số ví dụ điển hình cho kĩ thuật Ví dụ (ĐTTS_khốiA2003câu ΙΙ1 ) 21 Chuyên đề : Qui nạp toán học Giải hệ phương trình  x − x = y − y  2 y = x +  Điều kiện xy ≠ Hệ cho viết lại dạng :  −( x − y) x − y = xy  2 y = x3 +  Nhờ kĩ thuật viết lại đề , ta xác định hướng giải cho hệ xuất phát từ phương trình thứ hệ Ví dụ (ĐHCSNN_khối A2000) Cho hệ phương trình  x + xy + y = m +   xy ( x + y ) = m + : Giải hệ cho m=-3 Xác định m để hệ có nghiệm  x + y + xy = m + Hệ cho viết lại dạng :  xy x + y = m + )  ( Nhờ viết lại hệ mà ta đặt x+y= S, xy = P , Điều kiện S 2-4P ≥ , việc giải hệ pt có khăn Nói chung kĩ thuật giải toán , học sinh nên rèn luyện kĩ thuật để áp dụng trình giải tất dạng toán học sơ cấp Bài tập đề nghị Bài 1: CMR ∀n ∈ N * : 16 n − 15n − 1225 Bài 2: CMR ∀n ∈ N , u n = 13n − 1M6 ∀n ∈ N ,122n + + 11n + M 133 Bài 3: CMR Bài 4: CMR ∀n ∈ N : 4.3 n + + 32n − 36 64 Bài 5: CMR ∀n ∈ N : n + 2n chia hết cho Bài 6: CMR ( n + 1) ( n + ) ( 2n ) chia hết cho 1.3.5 ( 2n − 1) , n ∈ N 22 Chuyên đề : Qui nạp toán học Tài liệu tham khảo Doãn Minh Cường : Giới thiệu đề thi tuyển sinh đại học 20002001,NXBGD Hà Văn Chương : Tuyển tập 700 toán bất đẳng thức,NXB.Trẻ1998 Phan Đức Chính – Vũ Dương Thụy _Đào Tam – Lê Thống Nhất : Các giảng luyện thi môn toán ,tập hai NXBGD Đặng Đình Hanh :Tập đề tập quan hệ chia hết giành cho K53GH Trần Văn Kỉ : 460 toán bất đẳng thức,NXB.Trẻ TPHCM Ngô Thúc Lanh- Vũ Tuấn- Ngô Xuân Sơn : Đại số giải tích 11, Nhà xuất giáo dục1998 Võ Đại Mau :Phương pháp giải toán bất đẳng thức,NXB.Trẻ2000 Nguyễn Văn Mậu :Một số toán chọn lọc dãy số ,NXNGD Trần Phương : Phương pháp giải đề thi tuyển sinh môn toán, NXBGD 10 Nguyễn Tiến Quang :Bài tập số học, NXBGD 11 Bộ đề thi tuyển sinh đại học, Nhà xuất giáo dục 2001 12 Tuyển tập 30 năm tạp chí toán học tuổi trẻ, Nhà xuất giáo dục 13 Hai sách giáo khoa chỉnh lí hợp năm 2000 đại số giải tích 11, giải tích 12 Nhà xuất giáo dục 23 ... xin gửi địa email : tnngocthu@gmail.com Người thực Nguyễn Ngọc Thư Chuyên đề : Qui nạp toán học QUI NẠP Phương pháp qui nạp thực có hiệu lực với lớp toán chứng minh mệnh đề phụ thuộc vào số tự... đổi từ đt, bđt ứng với n = k+1, để dùng giả thiết qui nạp dẫn đến kết thúc toán mà ta biến đổi trực tiếp từ giả thiết qui nạp toán Đối với toán qui nạp để linh hoạt trình giải ta nên nhớ hai cách... pháp qui nạp để chứng minh đẳng thức VD1 : Chứng minh : với số tự nhiên n ≥ ,ta có : an – bn = (a – b)(a n – + a n – 2.b +… +a.b n -2 +b n– ) (1) Ta chứng minh đẳng thức (1) phương pháp qui nạp

Ngày đăng: 25/08/2017, 18:32

Từ khóa liên quan

Mục lục

  • TRƯỜNG ĐẠI HỌC SƯ PHẠM HÀ NỘI

  • KHOA TOÁN TIN

  • *********

  • Chuyªn ®Ò

  • QUI NẠP TOÁN HỌC

  • Lời mở đầu

  • Các dạng toán minh hoạ.

  • Dạng 1 : Dùng phương pháp qui nạp để chứng minh một đẳng thức .

Tài liệu cùng người dùng

  • Đang cập nhật ...

Tài liệu liên quan